PT14.S2.Q11: Rent-Controlled Ordinances

abbymajerusabbymajerus Core Member

I am not understanding how D is the correct answer. I actually chose D before blind review, mainly as a gut instinct to look back over later. Upon blind review, I changed my answer to B.
I now understand why B is wrong, as all we know is that many municipalities will choose rent controlled ordinances for short term gain, but there is no way of knowing whether or not they will choose to repeal them when the long-term disadvantages set in.

What I don't understand is how D is not subject to the exact same flaw. I understand that we know that many municipalities will choose rent control ordinances, which in theory would eventually lead to a shortage of rental units, but how can we properly infer that each municipality in that group either does or will actually make it to the point of experiencing that long-term disadvantage? Isn't it possible that they will all repeal the ordinances before shortages become a problem?

Maybe the reasoning is because while it is possible that they repeal before the shortages arise, we should assume that, because they are entirely motivated by the short-term gains, they would not repeal the ordinances before the shortages arise?

If there is a better way of reasoning this out please let me know, thanks! I am probably overthinking this one....

Comments

  • attorneysomervilleattorneysomerville Free Trial Member
    75 karma

    This is a "must be true" question, so you're going to have to PROVE the answer is true using ONLY the premises in the stimulus. If we find the right answer, it MUST BE TRUE, which means it CAN'T BE FALSE. So let's see how that works for answer choice D:

    D) In many municipalities there is, or eventually will be, a shortage of rental units.

    Let's make answer choice D false and see whether there's anything in the stimulus that gets in the way. Let's do this is study buddy form.

    Antonia: I think the answer is D, there will be a shortage of rental units in many cities.
    Trevor: A shortage of rental units? Huh uh! There won't be any shortage.
    Antonia: It says here rent-control ordinances have negative effects, and one of them is a shortage of rental units.
    Trevor: Yeah, UNLESS they are strictly temporary. Maybe they're all strictly temporary!
    Antonia: Except it says down here that in many municipalities, tenants get political power. They're not going to give up their low-cost rent!
    Trevor: Huh. Well, okay, so, but that doesn't mean there will be shortages.
    Antonia: Yes it does -- that's what the first sentence says!
    Trevor. Oh. Well, that doesn't mean D has to be right.
    Antonia: Yes it does! If it CAN'T be false, it MUST BE TRUE!

Sign In or Register to comment.